All Questions

Filter by
Sorted by
Tagged with
1 vote
1 answer
42 views

Are these Transformations of the Green's function equivalent?

The Green's function $G(E)$ can be constructed from the Hamiltonian $H$ $G(E) = [(E+i\epsilon)I - H]^{-1}$ where $I$ is the identity matrix. Say we want to perform a transformation into another basis ...
DJames's user avatar
  • 411
2 votes
1 answer
374 views

Spacetime as a medium for electromagnetic waves? [closed]

We typically break waves into two separate types mechanical waves and electromagnetic waves and treat them entirely differently based primarily on the observation that electromagnetic waves don't ...
user avatar
1 vote
1 answer
184 views

Approximation of Stable Orbits as Harmonic Oscillators

A textbook on classical mechanics I am currently reading considers the stable orbit (at $r_0$) of a body subject to the power law: $$\mathbf{F}(r)=-Kr^n\mathbf{\hat{r}},\quad n\in\mathbb{Z}$$ $$\...
Angelo Di Bella's user avatar
0 votes
1 answer
117 views

Does electric current through gases emit light?

Not electric spark or electric arc, just normal current through gases created by heating the air between cathode and anode as shown in the following video at 1:25. https://www.youtube.com/watch?v=pg0-...
Fizzics's user avatar
  • 59
0 votes
1 answer
96 views

Is it possible for a particle to have all the energy of the Isolated System of particles?

We have read the Fundamental postulate of statistical mechanics which says that : In a state of thermal equilibrium, All the accessible microstates of the system are equally probable. Suppose a ...
Young Kindaichi's user avatar
0 votes
0 answers
494 views

Wick's time ordering operator

I am working on Wolfgang Nolting's . And I feel confused about the time ordering operator.Below is how he introduced the Wick's time ordering operator. My understanding is that the $\epsilon$ in the ...
Tan Tixuan's user avatar
-2 votes
2 answers
50 views

Projection of ball. find initial velocity [closed]

A cannonball is fired at an angle of 33 degree relative to the horizontal as shown in the figure bellow. It is 1930 meters in the horizontal direction from the point of launch to the point where the ...
Ankkita Sharma's user avatar
3 votes
2 answers
572 views

If $T dS = dq$ only for reversible processes, why can I use it for non-reversible processes?

Assume some system with constant heat capacity $C_V$ at initial temperature $T_0$ is in thermal contact with a reservoir at temperature $T$. As is typically done, the net entropy increase in the ...
KvanteKaffe's user avatar
1 vote
1 answer
148 views

Where is the observer in AdS-Schwarzschild coordinates?

for an AdS-Schwarzschild black hole in 4d, the metric is $$ ds^2 = -f(r)dt^2 + \frac{dr^2}{f(r)} + r^2d\Omega^2 $$ where $f(r) = 1 + r^2/l^2 - C/r$. $l$ is the AdS length scale and $C$ is some ...
Johannes's user avatar
1 vote
3 answers
113 views

Postulate of a priori probability and harmonic oscillator

According to the fundamental postulate of a priori probability in Statistical Mechanics: An isolated system in equilibrium is equally likely to be in any of its accessible states. But for a ...
aneet kumar's user avatar
-4 votes
1 answer
85 views

Gravity between antimater and matter [duplicate]

Always considered antimater as negative mass so: $$m1=10kg(matter)$$ $$m2=-10kg(antimater)$$ $$displacement=r=10m$$ $$gravity =\frac{Gm1m2}{r^2}=-1N<0!HOW?$$
protectgoodlivingbeingask's user avatar
1 vote
1 answer
560 views

With air resistance, why does the horizontal range decrease and why is it steeper during the descent phase?

IN RELATION TO PROJECTILE MOTION I've been trying to grasp this concept but cannot figure it out. From what i understand, the x-component only has air resistance acting on it so it slows it down ...
sophia's user avatar
  • 13
7 votes
5 answers
2k views

Is information conserved in quantum mechanics (after wave function collapse)?

I have heard in popular science that there is a law of "conservation of information." Some times this is described as: for any event that happens, there is enough information to reconstruct ...
Steven Sagona's user avatar
0 votes
1 answer
410 views

Why ferroelectric material needs to be insulating?

Is it necessary for a ferroelectric material to be insulating? Is it possible for a metal to be ferroelectric?
Bikash Patra's user avatar
3 votes
3 answers
768 views

How do we know neutrons and electrons are distinct particles on the same scale as protons?

I'm aware my question may not even make sense when considering quantum physics, but please excuse my ignorance. We are taught in school that there are basically 3 particles that make up atoms (protons,...
neelsg's user avatar
  • 197
1 vote
0 answers
25 views

Is there one photon per wavelength of light? [duplicate]

This makes intuitive sense to me but I have never heard anyone confirm it. Is it one photon per wave length or does it approximate one wavelength or something else?
Shannon T's user avatar
  • 361
0 votes
2 answers
713 views

Confusion regarding calculation of Miller indices for planes

Let's calculate the Miller indices of the yellow plane. The intercepts are $(2,2,1)$ . Taking the reciprocals we get $1/2,1/2,1$ . Clearing the fractions I get the indices as $(1,1,2)$. Why then does ...
Kashmiri's user avatar
  • 1,220
16 votes
3 answers
4k views

"To come back to Earth...it can be five times the force of gravity" - video editor's mistake?

I've watched The Truth About Gravity With Professor Jim Al-Khalili | Gravity And Me | Spark where astronaut Chris Hadfield says at 3:55: To come back to Earth is violent Then after several seconds ...
Martian2020's user avatar
1 vote
0 answers
166 views

Non-markovian random walk. Elephant random walk

When one does the numerics for the usual random walk, one might use the transition matrix in order to get the probability ar time $t$ of the process. As expected, the asymptotic behavior yields a ...
user2820579's user avatar
1 vote
1 answer
346 views

Why is there no friction at the top and bottom of loops? [duplicate]

A beginner's question: Why does friction vanish at the top and bottom of loops? If friction works in the opposite direction of motion, then surely there must be some element of friction that is in the ...
Baksel's user avatar
  • 11
3 votes
0 answers
34 views

Hydrogen gas tube and the spectrum [duplicate]

Generally to detect the hydrogen spectrum people uses the hydrogen gas tube as a light source. My question is, since the gas in the tube is the hydrogen molecule $H_2$ why is the spectrum equal to the ...
amilton moreira's user avatar
-2 votes
1 answer
80 views

Is it feasible to measure one-way speed of light this way? [closed]

I know that this subject is pretty saturated even thought i waited a few days.Its a generalized theoretical expiremental set up to try and measure one way speed of light,i just want any input on this ...
kostarak's user avatar
2 votes
1 answer
69 views

Can we improve transmission of light through water?

Is there any way to add chemicals/elements to water to increase the penetration of light (em waves)?
Bingohank's user avatar
  • 135
0 votes
0 answers
302 views

What is the internal energy of water vapor? OR How many degrees of freedom does $\rm H_2O$ have?

Here are the "knowns" that I'm working with. Internal Energy for a monatomic gas is $$U = (3/2)nRT$$ For a diatomic gas its $$U = (5/2)nRT$$ All of my textbooks and online sources indicate ...
Physicator's user avatar
2 votes
3 answers
148 views

Measuring relative speeds in SR

Suppose I have a galactic ruler spanned from earth to Alpha Centauri. With marked units, so one can read off the distance starting with 0 at earth until 4 light-years at Alpha Centauri. Now according ...
martin's user avatar
  • 21
-1 votes
1 answer
73 views

Trying to solve second equation of motion without substituting $v=(at+v_i)$ [closed]

The derivation of $s = s_0 + v_0t + ½at^2$ starts with $v = dx/dt$, which is rewritten as $v dt = dx$ and then $v$ is replaced with $at+v_i$. Then it is integrated. Why is this substitution done, ...
unwanted's user avatar
0 votes
1 answer
63 views

What is the probing depth of the XAS spectroscopy at Iron edge?

I want to find the probing depth of the XAS spectroscopy at Iron edge, I found this paper: Magnetic circular dichroism study of Fe/Co/Cu (001) using electron yieldx-ray absorption spectroscopy with ...
j.doe's user avatar
  • 155
1 vote
3 answers
99 views

Why does the light beam move with the lightclock?

A flat platform is moving in a tunnel with velocity V relative to the tunnel. A stationary reference frame S1 is attached to the tunnel and a reference frame S2 is attached to the moving platform. On ...
Zamir's user avatar
  • 11
2 votes
1 answer
122 views

Entropy and time asymmetry

I have learned in Lagrangian Mechanics that the existence of time symmetries makes energy conserved. But from thermodynamics, we know that entropy introduces the forward arrow of time, i.e., the time ...
Little Physicist's user avatar
2 votes
1 answer
693 views

Connection between particle physics and weight diagrams

I have a hard time combining two topics that are often discussed in physics in a coherent way. In a lot of Introduction to particle physics-classes one will hear about "multiplets", which ...
Sito's user avatar
  • 1,205
1 vote
0 answers
107 views

Is the "preferred basis problem" about why things entangle in specific ways?

I know there are various definitions of the "preferred basis problem." I'm trying to understand what Wikipedia is saying here. ... a quantum state can equally well be described (e.g.) as ...
A_P's user avatar
  • 765
1 vote
0 answers
33 views

Transition between vibronic states and the meaning of transition dipole moment

Let's say we have a molecule which belongs to $D_{3h}$ point group. The IRREPs of inital and final states are as follows $A'_{1}$, $E'$. Hence, the IRREP for the dipole moment has to be $E'$ so that ...
jayjay's user avatar
  • 335
0 votes
2 answers
71 views

Energy of photon from superposition to ground state

Let an atom is in superposition of two eigen energy states E1 and E2. If it returns to the ground level E0 what energy should have the photon released in the process?
Mercury's user avatar
  • 591
0 votes
1 answer
130 views

Multiple-Slit Interference Clarification

Source: https://openstax.org/books/university-physics-volume-3/pages/3-3-multiple-slit-interference $dsin(\theta) = m \lambda$ (1) where $d \mbox{= spacing between slits}$ $\theta \mbox{= ...
WigbertPowrr's user avatar
0 votes
1 answer
209 views

Hamiltonian on qubit: necessarily induces rotation. What about the reflections? Are they impossible to implement?

Any Hamiltonian dynamic on a qubit is necessarily a rotation. I consider a two level system having a Hamiltonian $H$. Decomposing this Hamiltonian on the Pauli basis and using the fact it is Hermitian,...
StarBucK's user avatar
  • 1,350
0 votes
1 answer
856 views

Scalar field energy density

Considering a classical scalar field theory, I can find the canonical energy momentum tensor and if I calculate the $00$ component I get: $$T^{00}= \frac{1}{2} \dot \phi^2 + \frac{1}{2} (\partial_i) \...
TheoPhy's user avatar
  • 900
3 votes
1 answer
66 views

Is the every translationally invariant operator constructed from powers of the Laplacian operator?

Consider for simplicty 1d discrete chain - $x_1 \ldots x_n$, and let the field $\phi(x_i)$ reside on the sites of this chain. Operators $\mathcal{O}$ in this this theory are hermitian $n \times n$ ...
spiridon_the_sun_rotator's user avatar
2 votes
1 answer
291 views

Is a quasiparticle just an eigenstate of the Hamiltonian?

The description of quasiparticles seems to come in two flavors: Completely qualitatively, where it is simply said that different (quasi-)particles interact to "form" a quasiparticle, or ...
MrArsGravis's user avatar
0 votes
1 answer
280 views

Lorentz transformations and rest frame

So a Lorentz transformation leaves the internal product invariant in the Minkowski spacetime. Let's say I do a Lorentz transformation, either a boost or a spatial rotation and I choose the reference ...
TheoPhy's user avatar
  • 900
1 vote
1 answer
135 views

Invariant of supersymmetry?

Given two vectors in 3D superspace $(x_1^\mu,\theta_1^\alpha,\overline{\theta}_1^\alpha)$ and $(x_2^\mu,\theta_2^\alpha,\overline{\theta}_2^\alpha)$ I am trying to find a polynomial invariant under ...
user avatar
1 vote
1 answer
85 views

Heterodyne interferometry, an alternative?

I am trying to setup an interferometer to monitor displacements and tilts between two surfaces. I am quite struggling because I never studied optics deeply. I have found in this paper (https://...
Gianluca's user avatar
2 votes
2 answers
1k views

How much can water be compressed?

How much can liquid water be compressed? I mean the maximum achieved in practice and the maximum achievable in theory. Say for example one liter of water at 4 degrees Celsius. Can it be compressed to ...
Joe Jobs's user avatar
  • 233
0 votes
2 answers
552 views

How to compare several experimental measurements and their uncertainties?

I have a basic question about how to make the comparison of different measurements. For example, suppose I have measured the focal length of a convergent lens using three different methods, $A$ and $B$...
Invenietis's user avatar
0 votes
0 answers
46 views

Is temperature a Lorentz scalar [duplicate]

If I see a body at temperature $T$, will I see the same temperature in another frame under a Lorentz boost. And will the internal energy of a body also remain invariant under a Lorentz boost or not..
Shashaank's user avatar
  • 2,757
0 votes
2 answers
169 views

Why is the frame attached on the Earth non-inertial in General Relativity? ( neglecting rotations etc..)

The EEP says that freely falling frames are inertial. But suppose the freely falling frame is at some spacetime point. At that point you can by a suitable coordinate transformation reduce the metric ...
Shashaank's user avatar
  • 2,757
0 votes
3 answers
528 views

Why is the pressure in quantum infinite wells the derivative of total energy, not potential?

Consider some quantum well of infinite depth, that is, for some region $R$, $$ V(r)= \begin{cases} 0, \quad r \in R \\ \infty, \quad r \notin R \end{cases} $$ My professor said, without much ...
Nine Thousand's user avatar
0 votes
1 answer
263 views

Why is an RG fixed point scale invariant?

I cannot understand why people say RG fixed point is scale invariant. Scale invariant means the action $S[\phi]$ of the theory is invariant under scale transformation like $\phi(x)\to\lambda^{-\Delta}...
Jiahao Mao's user avatar
0 votes
1 answer
74 views

How to transform coordinates from heliocentric to a view from earth?

I'm trying to find and model the apparent motion of the retrograde motion of Mars as viewed on Earth, incorporating Lagrangian or Hamiltonian mechanics. For the lagrangian I have: $L = \frac{1}{2}m_{...
Cobra04's user avatar
0 votes
1 answer
318 views

Does discharging of capacitor create sound?

Today I found a wierd thing that as I removed the pin of the echo dot And after this I removed the adapter from the power supply and found a wierd buzz sound coming from the adapter but as I ...
Anonymous's user avatar
  • 868
0 votes
2 answers
66 views

Waves and guitar frets

I have a problem to solve for Physics III at university and I can't seem to understand how to solve this question. I have the fundamental frequency at $440$ Hz, $L=1$ m, and $ρ=0.002$ kg/m. After ...
christinaa's user avatar

15 30 50 per page